Specific Gynecology Course Objectives: Infections PA 603 Women's Health

Ace your homework & exams now with Quizwiz!

Question: How does the normal vaginal pH compare to the vaginal pH in postmenopausal women?

Answer: Postmenopausal pH is more basic. The normal pH in reproductive-aged women is 3.5-5.0, and the pH in postmenopausal women is often 5.5-6.8.

Question: What is an alternative antibiotic medication that can be used to treat syphilis in the setting of penicillin hypersensitivity?

Answer: Tetracycline or doxycycline.

Question: Which human papillomavirus (HPV) serotypes are most associated with cervical cancer?

Answer: Types 16 and 18.

tubo-ovarian abscess (TOA) + workup (ROSH)

A tubo-ovarian abscess (TOA) is an inflammatory mass involving the fallopian tubes, ovaries, or occasionally other pelvic organs. Most tubo-ovarian abscesses occur as a complication of pelvic inflammatory disease (PID). They are typically polymicrobial, with mixtures of aerobic and anaerobic organisms. Some common organisms isolated from cultures include Escherichia coli, aerobic streptococci, Bacteroides fragilis, Prevotella, and anaerobes such as Peptostreptococcus. The classic presentation of TOA includes acute lower abdominal pain, vaginal discharge, and systemic symptoms, such as fever and chills. This presentation is similar to moderate to severe cases of PID. A TOA is said to have ruptured when it is leaking contents into the abdominal cavity. Most patients with a ruptured TOA present with an acute abdomen and sepsis, which requires immediate surgical exploration. The diagnosis of TOA is made clinically based on history and an exam suggestive of PID and the presence of an inflammatory adnexal mass on pelvic imaging. The most common imaging modality is transvaginal ultrasound. However, computed tomography of the abdomen and pelvis is frequently used when gastrointestinal pathology, such as appendicitis or an abscess associated with inflammatory bowel disease, is within the differential. Treatment of a TOA involves antibiotics and sometimes drainage. The diagnosis is further supported by surgical findings if drainage is performed. Patients diagnosed with a TOA should be admitted for intravenous antibiotics. There are multiple appropriate antibiotic regimens, but cefoxitin and doxycycline or cefotetan and doxycycline are common choices. These are the same regimens typically used to treat patients admitted with PID. A patient with TOA meeting the following criteria should be treated initially with antibiotics alone: hemodynamically stable, no signs of an acute abdomen, abscess size < 7 cm in diameter, and premenopausal status. Furthermore, patients with an abscess > 7 cm who meet the rest of the above criteria may also undergo a trial of antibiotics alone. Those who are not improving or are worsening with antibiotics alone often require further intervention. Some patients can be treated with minimally invasive abscess drainage, while others require surgical treatment. Postmenopausal women with a suspected TOA are at higher risk for malignancy, thus surgical intervention is recommended initially for diagnostic and therapeutic purposes.

Question: What condition is characterized by recurrent episodes of meningitis caused by herpes simplex virus type 2 (HSV-2)?

Answer: Mollaret meningitis.

Question: What is the most common age group to develop a tubo-ovarian abscess?

Answer: 15-25 years of age.

Question: What is the name of the physical exam finding common in neurosyphilis where small pupils will accommodate near objects but do not react to bright light?

Answer: Argyll Robertson pupil.

uestion: What are other clinical manifestations of human papillomavirus (HPV) infection?

Answer: Cervical disease, anal cancer, nonanogenital mucosal disease (e.g., oral warts), and nongenital cutaneous disease (e.g., common cutaneous warts).

Question: What autosomal recessive syndrome characterized by intellectual disability and accelerated pathologic aging is an absolute contraindication to the use of metronidazole?

Answer: Cockayne syndrome.

Question: What are some side effects of imiquimod?

Answer: Dermal ulcer, abrasion, edema, application site reaction, skin erosion, and erythema of the skin.

Question: True or false: pregnant women with a history of genital herpes and prodromal symptoms but no active lesions at the time of labor should deliver vaginally.

Answer: False. Prodromal symptoms are an indication for cesarean delivery.

Question: What is the most common reason a vaginal culture is performed when vulvovaginal candidiasis is suspected?

Answer: If the patient's symptoms are resistant to azole therapy and Candida glabrata is suspected.

Atrophic vaginitis + classic findings (ROSH)

Atrophic vaginitis (also known as genitourinary syndrome of menopause) occurs from a decrease in serum estrogen concentration, usually due to menopause, either natural or surgical. It can also occur with premature ovarian insufficiency and temporarily during the postpartum period and lactation. Estrogen acts on receptors in the vagina, vulva, urethra, and trigone of the bladder to maintain optimal genital blood flow, epithelial surface lubrication, and the collagen content of the epithelium. With a reduction in estrogen, there are gradual vaginal changes. These include thinning of the superficial epithelial cells, loss of elasticity, loss of rugae in the vaginal canal, a reduction in vaginal secretions, an increase of the vaginal pH to > 5, shortening and narrowing of the vaginal canal, and increased subepithelial connective tissue. The vaginal tissue gradually becomes more susceptible to trauma, leading to bleeding, petechiae, and ulceration, which can occur even during regular intercourse. Patients present with complaints related to these changes, including painful intercourse, vaginal dryness, dyspareunia, decreased arousal and sexual desire, vulvar or vaginal bleeding, vulvovaginal itching and burning, vaginal discharge, and urinary tract symptoms. Physical exam must include a pelvic exam to rule out abnormal tissue growth. Care should be taken with a speculum exam not to cause trauma to the tissues. Classic findings include pale, dry, and smooth vaginal epithelium with loss of rugation. There can also be inflammation with patchy erythema, friability, bleeding, and petechiae. Urinary symptoms should prompt evaluation with a urinalysis, but otherwise, the diagnosis is made clinically. Symptoms progress without treatment and can become severe. First-line therapy is with lubricants and nonhormonal vaginal moisturizers that are most useful for patients with mild symptoms. Vaginal moisturizers are used 2 to 3 days per week, and lubricants are used during intercourse. Low-dose vaginal estrogen therapy is used as a second-line treatment for patients who do not respond to vaginal moisturizers and lubricants or for those patients with severe symptoms. Low-dose vaginal estrogen is available in inserts, rings, or cream. Sexual activity should be avoided for 12 hours after application. Progestin in low-dose preparations is not recommended to protect against endometrial cancer or hyperplasia. Low-dose vaginal estrogen preparations can be used indefinitely, but treatment should be based on the patient's symptoms and degree of vaginal atrophy. Systemic estrogen therapy is not recommended for the symptoms of atrophic vaginitis, but it can be combined with low-dose vaginal preparations in patients who have other menopausal symptoms requiring oral estrogen therapy and continue to have vaginal symptoms.

Bacterial vaginosis (BV) + Diagnostic Criteria (ROSH)

Bacterial vaginosis (BV) is a clinical condition caused by the facultative anaerobe Gardnerella vaginalis and is the most common cause of vaginal discharge in women of childbearing age. BV is characterized by several changes in the vaginal environment, including a shift of bacterial diversity (including facultative anaerobes) from the normal Lactobacillus species, a production of amines by the new bacterial microbiota, and an increase in vaginal pH to > 4.5. The lactobacilli species typically produce hydrogen peroxide, which prevents overgrowth of anaerobic organisms. If there is a relative decrease in lactobacilli species, the overproduction of proteolytic carboxylase enzymes by anaerobic organisms results in conversion of vaginal peptides to volatile, malodorous amines. Risk factors for BV include sexual activity, douching, and cigarette smoking. BV presents clinically with a homogeneous, thin, grayish-white discharge that has a malodorous, fishy smell. BV is not associated with dysuria, dyspareunia, vaginal pruritus, or vaginal discomfort. Complications of BV include preterm delivery, postpartum fever, and postabortal infection. BV is also a risk factor for human immunodeficiency virus and herpes simplex virus type 2. The diagnosis of BV may be based on the presence of the following four Amsel criteria: homogeneous, thin, grayish-white discharge coating the vaginal walls; vaginal pH > 4.5; positive whiff test on potassium hydroxide prep of vaginal discharge; and the presence of clue cells on wet prep. Clue cells are vaginal epithelial cells that have a stippled appearance due to coccobacilli that adhere to the edge of the cell. At least 20% of the epithelial cells should be clue cells in order for the test to be positive. A Gram stain of the vaginal discharge demonstrating the presence of Gardnerella vaginalis is the gold standard for diagnosing BV but is rarely performed outside of a research or hospital setting because it is time-consuming and requires the maintenance of Gram stain reagents.

Bacterial vaginosis (ROSH)

Bacterial vaginosis is a common cause of vaginal discharge in women of childbearing age. Bacterial vaginosis is due to a shift in the vaginal flora that causes a rise in the vaginal pH. The shift is typically due to a decrease in hydrogen peroxide- and lactic acid-producing lactobacilli and an increase in anaerobic bacteria. The risk factors for bacterial vaginosis include sexual activity, douching, and cigarette smoking. Most women with bacterial vaginosis are asymptomatic. Women who are symptomatic typically present with vaginal discharge or vaginal odor. The discharge is classically thin, white, and homogeneous. Bacterial vaginosis does not typically cause vaginal erythema or edema, dysuria, dyspareunia, vaginal pruritus, or vaginal burning. The diagnosis of bacterial vaginosis is based on the presence of at least three of the four Amsel criteria: homogeneous, thin, grayish-white discharge; vaginal pH > 4.5; positive whiff-amine testing; and clue cells on saline wet mount. The whiff-amine testing is positive when a fishy odor is present after 10% potassium hydroxide is added to a sample of vaginal discharge. Clue cells cannot be detected when microscopy is not available, so in these cases, the diagnosis of bacterial vaginosis can be made by physical exam, pH testing, and whiff-amine testing. Furthermore, there are polymerase chain reaction tests to detect the bacterial species, such as Gardnerella vaginalis, that frequently cause bacterial vaginosis. Pharmacologic therapy with metronidazole or clindamycin is recommended to treat symptomatic women with bacterial vaginosis. There are oral and topical regimens available. It is important to educate patients to not consume alcohol while taking metronidazole and to not use latex condoms when using clindamycin cream. Patients who drink alcohol while taking metronidazole can have a disulfiram-like reaction, including nausea and vomiting. Clindamycin cream can reduce the efficacy of latex condoms. Women with frequent recurrences of bacterial vaginosis may be treated with preventive therapy, either oral metronidazole or oral tinidazole daily for 1 week and vaginal boric acid daily for 3 weeks. Patients who are in remission after this regimen can then use metronidazole gel twice weekly for 4-6 months.

human papillomavirus [HPV] (ROSH)

Cervicitis is inflammation of the columnar epithelial cells of the endocervical gland of the uterine cervix that can be due to infectious (chlamydia, gonorrhea, human papillomavirus [HPV]) or noninfectious etiology. Human papillomavirus causes the most common sexually transmitted infection (STI) in the United States and can be classified, according to the risk of progression to cancer, into low-risk types (6,11, 42) and high-risk types (16, 18, 31, 33). Virtually all cases of cervical cancer are attributable to HPV infection, with HPV 16 accounting for approximately 50% of cases and HPV 18 for 20%. Low-risk types typically are not oncogenic and mainly cause genital warts (Condyloma acuminatum). High-risk types are more likely to progress to carcinoma because they produce E6 and E7 proteins that result in the destruction of p53 and RB genes, genes that are important in tumor suppressor pathways. Persistent cervical infection by high-risk HPV types can lead to cervical intraepithelial neoplasia (CIN), characterized by koilocytic change, disordered cellular maturation, nuclear atypia, and increased mitotic activity within the cervical epithelium. CIN is graded, based on the extent of epithelial cell involvement, into CIN I (involving one-third of the epithelium), CIN II (involving two-thirds of the epithelium), CIN III (involving more than two-thirds of epithelium), and carcinoma in situ (involving the entire thickness of the epithelium). Diagnosis is made by HPV viral typing. According to USPSTF, women between the ages of 21 and 29 years should undergo screening for cervical cancer every 3 years with cervical cytology alone. In women ages 30 and 65 years, screening should be done every 3 years with cervical cytology alone, every 5 years with high-risk HPV testing alone, or every 5 years with high-risk HPV testing in combination with cytology (cotesting). Most cases of HPV infections resolve spontaneously even without treatment within 2 years. Podophyllin, trichloroacetic acid, or podofilox can be used to treat genital warts. Surgical excision, cryotherapy, electrocautery, laser therapy, or intralesional interferon may be used for persistent infection. High-risk cases are treated by simple or radical hysterectomy, with or without chemotherapy and radiation treatment depending on the staging of the cancer. Recombinant human papillomavirus 9-valent vaccine prevents both genital and anal warts as well as cervical, vaginal, and vulvar cancers in women. It covers HPV types 6, 11, 16, 18, 31, 33, 45, 52, and 58.

Condylomata acuminata (ROSH)

Condylomata acuminata, or anogenital warts, are typically caused by HPV type 6 and 11. HPV is the most common sexually transmitted infection in the world. Sexual activity is the primary risk factor for anogenital HPV infection. Additional risk factors include immunosuppression and smoking. Condylomata acuminata (singular: condyloma acuminatum) typically develops on the vulva, penis, suprapubic area, perineum, and perianal skin. The lesions may be single or multiple and may vary in color (white, skin-colored, erythematous, violaceous, brown, or hyperpigmented). The warts are typically soft to palpation and appear as flat, dome-shaped, verrucous, and cauliflower-shaped. Patients are typically asymptomatic but may experience pruritus. Anogenital warts are diagnosed based on clinical appearance. Treatment includes self-therapy at home with imiquimod, podophyllotoxin, sinecatechins and office-based therapy with trichloroacetic acid, cryotherapy, or surgical removal.

Cervicitis HSV + regimen (ROSH)

Cervicitis, an acute inflammation of the glandular epithelium of the cervix, is a common presentation for sexually transmitted infections. The most common causes of sexually transmitted infection are Chlamydia trachomatis, Neisseria gonorrhoeae, Mycobacterium genitalium, Trichomonas vaginalis, and herpes simplex virus (HSV). Untreated sexually transmitted infections can affect sexual partners and, for the patient, may ascend, causing serious complications and increased risk of acquiring and transmitting HIV. Herpes simplex virus types 1 and 2 can both cause genital infections, but type 2 is more common. Patients may be asymptomatic, but they may also present with complaints of vaginal discharge, dyspareunia, intermenstrual or postcoital bleeding, pelvic or abdominal pain, and general vulvovaginal irritation. Urinary symptoms are not necessarily present unless the urethra is affected. Complaints of pain and fever are more concerning for pelvic inflammatory disease or HSV infection. On exam, vesicular lesions and ulcerations are especially suggestive of HSV. A friable cervix may also be noted. If cervical motion tenderness is present, pelvic inflammatory disease should be a clear concern. Even if these findings are not present, a clinical diagnosis of HSV should be considered and laboratory confirmation should be obtained. Active genital lesions should be unroofed and fluid sent for cell culture or PCR-based testing. PCR has the greatest overall sensitivity and specificity. HSV serology should also be considered in patients with a history of genital lesions or negative HSV culture or PCR. All patients presenting with their first episode of genital HSV should be started on oral antivirals as soon as possible. Recommended courses include acyclovir (400 mg tid for 7-10 days), famciclovir (250 mg tid for 7-10 days), and valacyclovir (1,000 mg bid for 7-10 days). There has been no significant difference noted between treatment efficacy of these three regimens, though acyclovir has the most supportive data, and valacyclovir is a simpler regimen to take (twice versus three times daily) but may be more expensive. While treatment is effective at decreasing the duration and severity of HSV disease, it does not eliminate latent virus, thus all patients are at risk of recurrence and may require repeated treatments or a chronic suppressive treatment approach. Many patients with sexually transmitted infections are at increased risk for other infections, pregnancy, and engaging in other risky behaviors, so all patients should have a complete workup based on history and physical findings, including diagnosis, empiric treatment, treatment of partners, and education on prevention, as indicated.

Chancroid (ROSH)

Chancroid is a sexually transmitted infection that is caused by Haemophilus ducreyi, a gram-negative, fastidious rod that clumps in long parallel strands producing the so-called "school of fish" or "railroad track" appearance. It is more prevalent in individuals between the ages of 15 and 19 years, with an incubation period of 2 to 10 days. It is rarely seen in the U.S. and is more common in resource-limited countries. Patients present with a painful genital ulcer that has a ragged border and a purulent base. The ulcer starts initially as a papule that progresses into a pustule and then into an ulcer within a few weeks. Patients may also report dysuria and dyspareunia, particularly in women. A physical exam reveals unilateral tender inguinal lymphadenopathy (buboes) that appears 1 to 2 weeks after onset of the ulcer. Diagnosis is often made clinically. No serologic tests exist for the causative organism, and the culture of the organism is not practical because the causative organism is fastidious and requires a special medium that is not readily available. Serologic tests (rapid plasma reagin, venereal disease research lab) should be obtained to exclude syphilis. Treatment of chancroid consists of antibiotic therapy and drainage of a fluctuant inguinal lymph node if present. Antibiotic options include azithromycin, ceftriaxone, or erythromycin.

Chlamydia trachomatis + Diagnosis + RF (ROSH)

Chlamydia trachomatis is an intracellular gram-negative bacterium and the most common bacterial cause of sexually transmitted infections. It is easily spread because most patients who are infected are asymptomatic. Screening for chlamydial infection is recommended for all sexually active women < 25 years of age and for women ≥ 25 years of age who are at increased risk for sexually transmitted infections. Symptoms associated with C. trachomatis cervicitis include changes in vaginal discharge, bleeding between menses, and postcoital bleeding. Physical examination in symptomatic patients may show mucopurulent endocervical discharge, endocervical bleeding, or edematous ectopy. Compared with gonococcal cervicitis, the discharge associated with chlamydial infections is less painful, less purulent, and more watery. Urinalysis that reveals pyuria with no organisms on Gram stain in a sexually active woman is highly associated with chlamydial infection because Chlamydia is difficult to Gram stain. The best test for diagnosis is a nucleic acid amplification test. Complications of C. trachomatis infections include pelvic inflammatory disease, ectopic pregnancy, and infertility.

Chlamydia (ROSH)

Chlamydia trachomatis is the most commonly reported bacterial sexually transmitted infection, most often occurring in patients < 25 years of age. It is an obligate intracellular bacteria with a similar structure to gram-negative bacteria. Risk factors include a high number of sexual partners and low socioeconomic status. Most patients are asymptomatic, but women can present with mucopurulent discharge and cervical inflammation. Patients can also present with salpingitis. Testing is either with urine, endocervical, or vaginal specimens. Patients who engage in oral or anal sex can be diagnosed by swabbing these areas as well. Nucleic acid amplification tests are the most sensitive and can be used for urine and endocervical specimens. However, cell culture, direct immunofluorescence, nucleic acid hybridization, and enzyme immunoassay tests are also used. Salpingitis and pelvic inflammatory disease are complications of genital infection. Treatment is with azithromycin 1 gram PO as a single dose, doxycycline 100 mg PO twice daily for 7 days plus ceftriaxone 500 mg IM as a single dose, or cefixime 400 mg PO as a single dose. Coverage for N. gonorrhoeae is recommended. Pregnant patients are treated with amoxicillin 500 mg PO three times per day for 7 days with repeat testing 3-4 weeks after therapy. Annual screening is recommended for patients < 25 years of age with risk factors such as multiple or new sexual partners. Patients who test positive should inform all sexual partners in the last 60 days for evaluation, testing, and treatment.

Condyloma acuminata (ROSH)

Condyloma acuminata (genital warts) is a manifestation of anogenital infection with human papillomavirus infection. Human papillomavirus types 6 and 11 are responsible for most cases of condyloma acuminata. Human papillomavirus is almost always acquired through sexual activity. Immunosuppression (HIV infection, uncontrolled diabetes mellitus, immunosuppressive medications) increases the risk of acquiring condyloma acuminata and the likelihood of being refractory to treatment. The incubation period of condyloma acuminata varies widely from 3 weeks to 8 months. Condyloma acuminata occurs most often on the vulva, penis, groin, perineum, perianal skin, and suprapubic skin. Less frequently, lesions may occur on the vaginal mucosa, cervix, anal canal, and urethra. The appearance of condyloma varies widely and may include white, skin-colored, or brown lesions that are flat, dome-shaped, verrucous, fungating, smooth, cauliflower, and pedunculated. The size varies from as small as 1 mm up to several centimeters. The expected clinical course varies. The lesions could increase in size and number or resolve spontaneously. Furthermore, resolution of the lesions does not always mean the virus has been eradicated. Malignant transformation (genital warts turning into malignancy) is generally rare but may occur in immunocompromised patients. The diagnosis of condyloma acuminata is made based on clinical exam. Biopsy is not usually necessary but can be performed in atypical cases. Although condyloma acuminata may resolve spontaneously, treatment should be offered to all patients. The treatments are divided into patient-applied and clinician-administered therapies. Patients treated with patient-applied therapies must be able to reach all lesions and be willing to adhere to several weeks of treatment. The patient-applied options include imiquimod, podophyllotoxin, and sinecatechins. The clinician-administered options include cryotherapy, surgical excision, and electrosurgery. Patients are often treated with patient-applied therapy during the intervals between clinician-administered therapy.

Chlamydia Cervicitis RAPID REVIEW

Diagnosis is made by nucleic acid amplification testing (NAAT) Most commonly caused by Chlamydia trachomatis Treatment is doxycycline (100 mg BID x 7 days), azithromycin should be used in pregnancy Reinfection testing after treatment: -Nonpregnant: three months after treatment or at the first visit in the 12 months after treatment -Pregnant: four weeks after treatment Most commonly reported sexually transmitted disease in the United States Empirically treat for concomitant gonorrhea The USPSTF recommends routine screening for chlamydia and gonorrhea in sexually active women < 25 years of age and in women age ≥ 25 years who are at increased risk

Genital herpes simplex virus (HSV) (ROSH)

Genital herpes simplex virus (HSV) is a common sexually transmitted infection, typically caused by HSV type 1 or HSV type 2. Historically, HSV-1 has been associated with oral herpes (i.e., cold sores), and HSV-2 has been associated with genital herpes, but either HSV type can cause lesions in the oral and genital lesions. There has been an increase over the last few decades of HSV-1 genital herpes in younger, sexually active adults and men who have sex with men. Genital HSV infections are classified as primary, nonprimary, or recurrent. Primary genital infections are characterized by an HSV-1 or HSV-2 infection in a patient without preexisting HSV antibodies to either type. Nonprimary genital infections are characterized by an infection with HSV-1 in a patient with preexisting HSV-2 antibodies or infection with HSV-2 in a patient with preexisting HSV-1 antibodies. A recurrent genital infection is characterized by a reactivation of the HSV type to which the patient has already developed antibodies. Primary genital infections may be asymptomatic or present with multiple painful or pruritic genital ulcers, tender inguinal lymphadenopathy, fever, headache, malaise, myalgias, or dysuria. The incubation period after exposure is 2-12 days, and the average time to resolution is 19 days. Nonprimary infections are associated with fewer herpetic lesions and milder systemic symptoms. Symptoms associated with recurrent infections are less severe than either primary or nonprimary infections, and the average time to resolution is 10 days. Extragenital complications associated with genital herpes include neurologic involvement (e.g., aseptic meningitis, lumbosacral radiculitis, urinary bladder retention, autonomic nervous system dysfunction) and proctitis. Genital herpes, particularly primary and nonprimary genital herpes, can be a major complication in pregnant women because it can be transmitted to the fetus during vaginal delivery and is associated with increased fetal morbidity and mortality. HSV genital lesions appear as grouped vesicles on an erythematous base that progress into painful shallow ulcers. Viral culture is the standard diagnostic test, and active lesions should be unroofed to obtain vesicular fluid for culture. Polymerase chain reaction (PCR) testing of lesions may be used with suspected HSV. Direct fluorescent antibody and serologic testing may also be used. The presence of multinucleated giant cells on Tzanck smear is a common diagnostic finding associated with HSV infection but is not used clinically because of low sensitivity and specificity. First-line treatment for genital HSV infection is antiviral therapy such as acyclovir, valacyclovir, or famciclovir. Antiviral therapy should be started within 72 hours of lesion appearance. Patients with six or more HSV outbreaks each year or immunocompromised patients who wish to reduce the risk of HSV transmission can be given chronic suppressive therapy. Topical antiviral therapy is not indicated for genital HSV infection.

Gonorrhea (ROSH)

Gonorrhea is an infection caused by the gram-negative diplococcus organism Neisseria gonorrhoeae. Gonorrhea infections affect the cervix (cervicitis) most frequently in women, but it can also affect the urethra (urethritis). Symptoms and signs of gonorrheal cervicitis include mucopurulent vaginal discharge and vaginal pruritus. However, these findings are nonspecific and most cervical gonorrhea infections in female patients are asymptomatic. Physical exam findings may include vaginal discharge and a friable cervix, but the exam may also be normal. Complications of gonorrhea include pelvic inflammatory disease, infertility, ectopic pregnancy, and chronic pelvic pain. Diagnostic testing is required to confirm the diagnosis of gonorrhea since the symptoms and signs lack specificity. Nucleic acid amplification testing is the best diagnostic test. Gonorrhea cervical infections are treated with antibiotics. The recommended antibiotic regimen to treat gonorrheal cervicitis in a pregnant woman is a combination of ceftriaxone (500 mg intramuscularly) and azithromycin (1 g orally). Patients treated with this combination of antibiotics do not require testing to confirm cure (test of cure). However, patients with gonorrhea infection are at high risk for recurrent infections, thus it is recommended to repeat testing at 3 months. It is important for the sexual partners of patients with gonorrheal cervicitis to be treated as well. The preferred method for sexual partners to be treated is by testing and evaluation. However, expedited partner therapy is also a reasonable approach when evaluation is not possible.

For each of the following conditions, students should describe the underlying pathophysiology, presenting signs and symptoms, basic epidemiology, differential diagnosis, associated systemic disorders (if indicated), appropriate diagnostic studies, clinical intervention, pharmaceutical therapies, and health maintenance concerns: Cervicitis - Herpes simplex

HERPES SIMPLEX VIRUS 1 TRANSMISSION •Direct contact with contaminated saliva or other infected bodily secretions (eg, mouth to mouth contact, shared drinkware). ->90% of people infected worldwide PATHOPHYSIOLOGY •Direct contact at mucosal or skin sites cause viral entry into the epidermis until it reaches the sensory & autonomic nerve endings. -Resides in trigeminal ganglion Primary lesions: •Most primary infections are asymptomatic but may cause tonsillopharyngitis in adults and gingivostomatitis in children. •Herpetic whitlow: can occur in dentists and health care workers exposed to infected secretions. (infection of fingers from inoculation of abraded skin; vesicular and pustular lesions with erythema, pain, drainage) Secondary lesions: •Herpes labialis (cold sore): reactivation of latent infection in ganglion neurons characterized by prodromal symptoms (pruritus, burning, tingling or pain) within 24 hours followed by the development of grouped vesicles on an erythematous base that undergoes crusting prior to healing. DIAGNOSIS •PCR: test of choice (most sensitive and specific). HSV-1 serology gold standard. •Viral cultures, direct fluorescent antibody and ELISA; Tzanck smear (nonspecific finding of multinucleated giant cells). MANAGEMENT •Orolabial: oral Valacyclovir (2g bid x 1 day). Acyclovir is an alternative. •Chronic suppression may be needed for recurrent outbreaks. COMPLICATIONS of HSV-1 -Herpes encephalitis (fever, AMS, obtundation, seizures) and HSV-keratitis (tearing, pain, erythema, conjunctival swelling) GENITAL HERPES •Most cases of recurrent genital herpes are caused by Herpes simplex 2. HSV-1 less common cause. •Seen in about 25% of the population in the US. TRANSMISSION •Sexually transmitted via direct close contact with infected lesions. •The virus can enter and stay dormant in the sensory (sacral) nerve ganglion where it can become activated. CLINICAL MANIFESTATIONS •Painful genital ulcers often preceded by prodromal symptoms (eg, burning, paresthesias, numbness, pruritis). Dysuria, fever PHYSICAL EXAMINATION •Multiple, shallow, tender ulcers. Grouped vesicles on an erythematous base, inguinal lymphadenopathy. DIAGNOSIS •PCR is the test of choice (most sensitive and specific). •HSV-1 serology gold standard (not as sensitive or specific as PC). Viral cultures, direct fluorescent antibody. •Tzanck smear: multinucleated giant cells (intranuclear eosinophilic Cowdry A inclusions). Classic but not specific (can be seen with HSV1, HSV 2 and VZV). MANAGEMENT •Acyclovir, Valacyclovir, Famciclovir

Herpes simplex virus (ROSH)

Herpes simplex virus (HSV) is a common virus that can cause oral or genital manifestations and may be transmitted by direct sexual contact. Transmission is more likely in individuals with active lesions, but asymptomatic individuals may shed the virus. There are two subtypes of herpes simplex virus called HSV-1 (herpes simplex virus type 1) and HSV-2 (herpes simplex virus type 2). Historically, HSV-1 was associated with oral lesions, while HSV-2 was associated with genital lesions. However, each subtype can cause oral or genital symptoms. The classic manifestations of genital herpes simplex virus are painful vesicular lesions on an erythematous base that eventually rupture and form ulcers. Individuals with genital herpes simplex virus often have recurrent outbreaks. The clinical manifestations are frequently classified into three types of infection: primary infection, nonprimary first episode, and recurrent infection. In order to distinguish between these types of infection, the diagnosis of genital herpes should be confirmed with laboratory testing, and serologic studies must be sent to assess antibodies to HSV-1 and HSV-2. Primary infection occurs when laboratory testing confirms genital herpes (caused by HSV-1 or HSV-2) but the patient has no antibodies to HSV-1 or HSV-2. Primary infection indicates that this is the first infection (asymptomatic or symptomatic) that the individual has had to herpes simplex virus. Primary infection causes the most severe manifestations. The manifestations include painful genital fluid-filled vesicular lesions that ulcerate, dysuria, tender local lymphadenopathy, and systemic manifestations, such as fever, malaise, and headache. Nonprimary first episode occurs when laboratory evidence confirms genital herpes but the individual has antibodies to the other herpes simplex virus subtype. For example, when HSV-2 is determined to have caused the genital herpes outbreak but the individual has antibodies to HSV-1 from a previous infection. The manifestations of nonprimary first episode are less severe compared to primary infection. There are fewer lesions and less systemic symptoms. Recurrent infection is diagnosed when the individual has a genital HSV infection confirmed with laboratory testing and also has serum antibodies to the same subtype. Interestingly, it is possible for an individual to have a recurrent infection without prior symptomatic episodes. Recurrent infections are common and less severe than primary or nonprimary first episode infections. Systemic symptoms are absent, and the lesions tend to heal quicker. Viral culture or polymerase chain reaction testing are the preferred methods to diagnose genital herpes simplex virus. Genital herpes simplex virus infection is treated with antiviral medications, such as acyclovir or valacyclovir. The duration of treatment varies according to the underlying type of infection (primary, nonprimary first episode, or recurrent). Patients with frequent recurrent infections are sometimes treated with daily suppressive therapy rather than episodic therapy. It is important to provide patient education regarding the risk of transmission of herpes simplex virus and the recurrent nature of the virus.

Pelvic Inflammatory Disease (PID) RAPID REVIEW

History of multiple sexual partners or unprotected intercourse Lower abdominal pain, cervical motion tenderness, painful sexual intercourse PE will show mucopurulent cervical discharge Most commonly caused by Chlamydia trachomatis Outpatient treatment is ceftriaxone + doxycycline + metronidazole Fitz-Hugh-Curtis syndrome: perihepatitis + PID

Tubo-Ovarian Abscess RAPID REVIEW

History of pelvic inflammatory disease (PID) Lower abdominal pain, fever, vaginal discharge PE will show unilateral adnexal tenderness Diagnosis is made by ultrasound Most commonly caused by a complication of pelvic inflammatory disease Treatment is intravenous antibiotics, surgical drainage, or both

Question: Which condition refers to a fallopian tube that is blocked with fluid?

Hydrosalpinx

Lymphogranuloma venereum + RF + Eval (ROSH)

Lymphogranuloma venereum is a genital ulcer disease caused by specific serotypes of Chlamydia trachomatis. Historically, it has been most common in tropical and subtropical climates, such as West and East Africa, India, parts of Southeast Asia, and the Caribbean. However, the prevalence of lymphogranuloma venereum has increased in the United States and Western Europe recently in men who have sex with men. There are three stages of lymphogranuloma infection: primary infection, secondary infection, and late lymphogranuloma venereum. Primary infection presents with a painless and small genital ulcer at the site of inoculation. The painless ulcer resolves spontaneously within a few days and thus often goes unnoticed. Secondary infection occurs 2 to 6 weeks following the primary infection. The secondary stage is due to inflammation from direct extension to the superficial and deep inguinal or femoral lymph nodes. The secondary stage may manifest as an inguinal syndrome or as anorectal symptoms. The unilateral painful inguinal lymph nodes associated with lymphogranuloma venereum are often called buboes. The inguinal syndrome of lymphogranuloma venereum is less common in women because the vaginal and cervical lymph nodes drain predominantly to the retroperitoneal area. The secondary stage of lymphogranuloma venereum may lead to severe inflammation, causing systemic symptoms. Anorectal symptoms due to lymphogranuloma venereum include rectal discharge, tenesmus, anal pain, and constipation. Late findings of lymphogranuloma (typically in untreated cases) include strictures or fibrosis of the genitourinary tract, infertility, genital elephantiasis, and esthiomene, which is defined as destruction of the genitalia. Lymphogranuloma diagnosis is difficult but can be confirmed with laboratory testing, which may include cultures, serology, or nucleic acid amplification testing (most accurate). The recommended treatment consists of a 21-day course of doxycycline. The buboes are sometimes drained for symptomatic relief. It is important to have all sexual partners of the patient evaluated and treated and to have the patient tested for hepatitis B, hepatitis C, and HIV infection.

For each of the following conditions, students should describe the underlying pathophysiology, presenting signs and symptoms, basic epidemiology, differential diagnosis, associated systemic disorders (if indicated), appropriate diagnostic studies, clinical intervention, pharmaceutical therapies, and health maintenance concerns: Cervicitis - Human papilloma virus

MC: condylomata acuminatum CLINICAL SYMPTOMS AND SIGNS 1. Most asymptomatic 2. Flesh-colored, papillary exophytic lesions on genitalia (vulva and perianal skin) 3. External warts - small bumps or flat, verrucous pedunculated 4. Vulva and vaginal warts Diagnosis 1. RPR/VRDL-r/o syphilis 2. HIV 3. HPV viral culture 4. Shave or punch biopsy confirmatory tREAtMENt 1. Most resolve spontaneously in 2 years 2. Podophyllin or tca - provider 3. Podofilox - patient 4. Surgery - cryotherapy, excision, electrocautery 5. Vaccine (below) HPV VACCINATION •HPV types 16 and 18 cause ~70% of all Cervical cancers worldwide and nearly 90 percent of anal cancers, as well as a significant proportion of oropharyngeal cancer, vulvar, vaginal cancer, and penile cancer. •HPV types 6 and 11 cause ~90% of genital warts. INDICATIONS: •Females: given in women age 11 up to 26 years of age. VACCINES: •Gardasil 9 (preferred): targets the same as Gardasil (6, 11, 16, 18) as well as HPV types 31, 33, 45, 52, & 58. •Gardasil: quadrivalent HPV vaccine that targets HPV 6, 11, 16, & 18. DOSING: <15y: 2 doses of HPV vaccine at least 6 months apart. •15y or older or immunocompromised: should receive 3 doses over a minimum of 6 months. Classically administered at day 0, at 2 month & at 6 months. Minimum interval between first 2 doses is 4 weeks, minimum interval between the second & 3rd is 12 weeks. •HPV vaccine is contraindicated if pregnant or lactating.

Chancroid RAPID REVIEW

Risk factors: sexually active Sx: painful genital ulcers PE: papule evolves to a pustule which ulcerates, ulcers on an erythematous base covered by a gray or yellow purulent exudate and painful lymphadenopathy (bubo) Caused by Haemophilus ducreyi Tx: ceftriaxone 250 mg IM or azithromycin 1 g oral

Neisseria gonorrhoeae + complications (ROSH)

Neisseria gonorrhoeae is a gram-negative diplococcus that can infect the urethra, cervix, anal canal, and pharynx. It is transmitted through sexual activities and is reported to the health department. After invading through the columnar and transitional epithelium in the genitourinary tract, it can enter the upper reproductive tract and cause salpingitis. The incubation period is typically 3-5 days. Many patients are asymptomatic, but typical symptoms that can occur are vaginal discharge, dysuria, urinary frequency, and rectal discomfort. Patients may also complain of itching or burning. Gonorrhea can be transmitted during delivery, causing ophthalmia neonatorum. Thus all newborns should receive erythromycin ophthalmic ointment 0.5% to each eye as a single application after delivery. The diagnosis of gonorrhea should be confirmed with endocervical, vaginal, or urine specimens using culture, nucleic acid amplification tests, or nucleic acid hybridization tests. If a patient fails initial treatment, culture with antimicrobial sensitivity testing is recommended. Treatment for concomitant chlamydia infection is recommended, and patients should be given either ceftriaxone 500 mg IM with doxycycline 100 mg PO twice daily for 7 days. Salpingitis, which can result in tubal scarring, is the major complication of this infection. Patients should abstain from sexual activities until at least 7 days after the initiation of therapy and they no longer have symptoms. Condoms will protect against gonorrhea. Sexual partners of the patient with whom they have had sexual contact in the last 60 days prior to diagnosis should also be treated.

Condyloma Acuminata RAPID REVIEW

Patient presents with genital lesions PE will show cauliflower-like lesion Most commonly caused by HPV 6 & 11 Most common STI

Bacterial Vaginosis RAPID REVIEW

Patient presents with malodorous vaginal discharge PE will show thin, gray or white discharge Labs will show pH > 4.5, clue cells Diagnosis is made by potassium hydroxide smear → fishy odor, whiff test, Amsel criteria Most commonly detected bacteria is Gardnerella vaginalis (usually due to decrease in Lactobacillus sp) Treatment is metronidazole

tubo-ovarian abscess (ROSH)

Pelvic inflammatory disease (PID) can be complicated by a number of processes, including endometriosis, salpingitis, oophoritis, peritonitis, perihepatitis, and tubo-ovarian abscess. Risk factors for tubo-ovarian abscess are the same as those for PID and include age of 15-25 years, multiple sexual partners, and sexually transmitted infections. As a complication of PID, tubo-ovarian abscess is uncommon but life-threatening. The pathogenesis of tubo-ovarian abscesses is not well understood but is thought to involve uncontrolled inflammation of the adnexal system and may also be due to inflammation of the bowel or hematologic spread. Bacteria found in tubo-ovarian abscesses are usually polymicrobial and similar to those found in patients with uncomplicated PID. While PID is the most understood precursor to tubo-ovarian abscess, there may be other causes. In particular, malignancy should be considered for postmenopausal women presenting with signs of tubo-ovarian abscess. Women with tubo-ovarian abscess present similarly to those presenting for PID, including fever, chills, acute lower abdominal pain, and vaginal discharge. A septic appearance is concerning for ruptured abscess. Initial workup should include history and pelvic examination, pregnancy test, testing for sexually transmitted infections, CBC, and CRP. Elevated white blood cell count and elevated inflammatory markers are more associated with tubo-ovarian abscess than PID alone. Hospital admission is also indicated, as parenteral antibiotics will need to be started and additional imaging considered. A diagnosis of PID should concurrently lead to decision-making around whether additional evaluation for tubo-ovarian abscess is warranted. The best imaging modalities include pelvic ultrasound and pelvic CT. Women who are acutely ill, who have significant abdominopelvic tenderness preventing a complete pelvic exam, or who have an adnexal mass noted on exam should be imaged. Women who have not improved after 48 hours of broad-spectrum antibiotics may need repeat imaging even if the first study was negative. Most women can be treated with antibiotics alone, provided they are hemodynamically stable, their abscesses are < 7 cm in diameter, and there is no evidence of rupture. In women who have larger abscesses or concern for rupture or who are not responding to antimicrobial therapy, surgical intervention should be considered in addition to antibiotics. A minimum of 14 days of antibiotics is routine, though many recommend treatment until the abscess is completely resolved, sometimes 4-6 weeks. Patients responding to parenteral treatment can complete their course of oral antibiotics when they meet discharge criteria. Since the advent of broad-spectrum antibiotics, the mortality rate for nonruptured tubo-ovarian abscesses has fallen from half of all cases to nearly zero.

PID + Complications (ROSH)

Pelvic inflammatory disease (PID) is caused by a sexually transmitted infection that affects the upper genital tract structures and is most commonly seen in women 15-25 years of age. Risk factors include multiple sexual partners, inconsistent condom use, and previous PID. PID is usually caused by a polymicrobial infection of aerobic and anaerobic bacteria. The most common bacterial pathogens are Chlamydia trachomatis and Neisseria gonorrhoeae. Clinical features of PID include bilateral lower abdominal and pelvic pain, nausea, headache, and low back pain. Fever, vomiting, urinary frequency, and abnormal vaginal discharge may also be present. Physical examination will reveal cervical motion tenderness, purulent endocervical or vaginal discharge, and occasionally uterine or adnexal tenderness. Laboratory testing should include a pregnancy test, microscopic examination of vaginal discharge, nucleic acid amplification testing for C. trachomatis and N. gonorrhoeae, HIV screening, and syphilis screening. Transvaginal ultrasound may be used to evaluate for a tubo-ovarian abscess, especially if there is unilateral adnexal tenderness. Diagnosis is typically made based on clinical presentation and associated findings on pelvic examination in a young, sexually active woman. Outpatient treatment for PID includes empiric antibiotic coverage with ceftriaxone 500 mg IM in a single dose (1,000 mg if the patient is ≥ 150 kg), doxycycline 100 mg PO bid for 14 days, and metronidazole 500 mg PO bid for 14 days with follow-up in 72 hours. Patients with PID with high fever, inability to tolerate oral fluid intake, severe abdominal pain, or suspected abscess or pregnancy should be hospitalized and treated with cefoxitin 2 g IV every 6 hours (or cefotetan 2 g IV every 12 hours) and doxycycline 100 mg PO or IV every 12 hours. Patients with PID should refrain from sexual activity until therapy is completed, and all partners should be evaluated and treated. PID results in scarring and adhesions in the genital tract as tissues heal, which could eventually result in several complications. Additionally, delay in treatment and repeated infections increase the risk of complications. Major complications of PID include tubo-ovarian abscess, chronic pelvic pain, infertility, and ectopic pregnancy.

Lymphogranuloma Venereum RAPID REVIEW

Primarily seen in men who have sex with men History of recent travel to tropical and subtropical areas of the world Small, shallow painless genital ulcer PE will show tender inguinal or femoral lymphadenopathy Most commonly caused by Chlamydia trachomatis Treatment is doxycycline

Syphilis RAPID REVIEW

Primary: painless chancre Secondary: lymphadenopathy, condyloma lata, rash on palms and soles Tertiary: gummas VDRL and RPR positive 1-4 weeks after infection Primary or secondary: IM benzathine penicillin G, 1 dose Tertiary: IM benzathine penicillin G qwk for 3 weeks

Vulvovaginal Candidiasis RAPID REVIEW

Risk factors: diabetes, HIV, recent antibiotic use, steroid use, pregnancy, immunosuppression Sx: vulvar pruritus, dysuria, dyspareunia PE: white, cottage cheese-like discharge Labs: normal pH < 4.5, wet prep: budding yeast, pseudohyphae, hyphae Most commonly caused by Candida albicans Tx: topical azoles, oral fluconazole

Genitourinary Syndrome of Menopause (Atrophic Vaginitis) RAPID REVIEW

Risk factors: natural or surgical menopause, antiestrogenic drugs Sx: dyspareunia, vulvar and vaginal dryness, bleeding, itching PE: pale, dry, shiny epithelium Caused by a decrease in estrogen Tx: lubricants, moisturizers, topical estrogen

lymphogranuloma venereum (ROSH)

Select serotypes of Chlamydia trachomatis can cause a genital ulcer disease known as lymphogranuloma venereum that affects lymphatic tissue. It is primarily associated with men who have sex with men and is more prevalent in tropical and subtropical areas. Recently, it has been increasing in incidence in more temperate climates and likely affects both sexes equally, though it is more commonly reported in men because the early clinical presentation in men is more obvious. Women usually present in later stages of disease when they develop complications. The process appears to extend directly from primary sites of infection to draining lymph nodes. Because of the lymph node involvement, it can clinically be confused with cat-scratch disease. The primary phase of infection, usually manifested as genital ulcers or mucosal inflammation, can be easily missed, as it usually heals spontaneously within a few days. In the secondary phase weeks later, direct extension to regional lymph nodes can cause painful inguinal lymph nodes or the sensation of inflammatory masses in the rectum and retroperitoneum. Because vaginal lymph nodes drain to the deep iliac or perirectal nodes, women may experience lower-abdominal or low-back pain, especially if they have had receptive anal sex, which could lead to primary rectal infection. Without treatment, the chronic inflammatory response and destruction of tissues can lead to the irreversible formation of perirectal abscesses, fistulas, strictures, rectal stenosis, lymphorroids (hemorrhoid-like swellings of obstructed lymphatics), chronic edema, sclerosing fibrosis, disfiguring elephantitis and esthiomene, and frozen pelvis. Lymphogranuloma venereum is difficult to diagnose clinically because the presentation tends to be nonspecific. However, nucleic acid amplification testing has good sensitivity and specificity to detect Chlamydia trachomatis and ultimately to differentiate lymphogranuloma venereum from non-lymphogranuloma venereum serotypes. Serology can be helpful in some settings but cannot distinguish current from prior lymphogranuloma venereum infections. Accurate, early diagnosis and effective treatment are essential to completing treatment of infection and preventing permanent damage to tissues. The recommended treatment for all nonpregnant patients is doxycycline 100 mg oral twice daily for 21 days. If there is evidence of more disseminated or system infection, a longer duration of treatment may be indicated. Shorter courses and other regimens have less evidence of complete treatment without additional sequelae of disease. Alternative regimens for pregnant and lactating women, or those who cannot take doxycycline, have less supportive data and have inferior treatment outcomes. However, azithromycin and erythromycin are options in these cases. In addition to treatment of the patient, all sexual partners with contact in the 60 days before or after infection or symptom onset should also be treated. Both the patient and sexual contacts should be thoroughly evaluated for other sexually transmitted infections, including HIV and syphilis, and counseled on protection and prevention of further infection and risky behaviors.

Trichomoniasis RAPID REVIEW

Sx: malodorous vaginal discharge, pruritus, dysuria in sexually active person PE: frothy, green or yellow discharge, petechiae on cervical mucosa: strawberry cervix Dx: vaginal pH > 4.5 Wet mount: flagellated, motile, pear-shaped organism Nucleic acid amplification and rapid antigen tests are more sensitive Causal organism: Trichomonas vaginalis Tx: oral metronidazole

Genital Herpes Simplex RAPID REVIEW

Sx: painful genital rash, may be asymptomatic PE: grouped erythematous shallow cluster of vesicles and lymphadenopathy Labs: multinucleated giant cells on Tzanck smear (poor sensitivity) Dx: tissue PCR or viral culture Most commonly caused by herpes simplex virus (HSV) type 2, but HSV-1 infections are increasing in frequency Tx: acyclovir -Pregnancy: acyclovir or valacyclovir for 7-10 days after primary infection and from 36 weeks to delivery

Primary Syphilis RAPID REVIEW

Sx: painless ulcer that forms 3 weeks post sexual activity and disappears in 3-6 weeks PE: vaginal, anal, or oral chancre - painless, punched-out lesion with a raised margin, lymphadenopathy Dx: darkfield microscopy, RPR/VDRL confirmed by FTA-ABS Caused by spirochete Treponema pallidum Tx: single IM injection of benzathine penicillin Mnemonic: syphilis is painless ulcer

Syphilis (primary) (ROSH)

Syphilis is a highly contagious disease caused by the spirochete Treponema pallidum. The infection enters the body at the site of physical contact with an infected individual and then spreads via the lymph system. Syphilis is classified by four types: primary, secondary, latent, and tertiary syphilis. Symptoms most commonly begin roughly 3 weeks after the initial infection, but the inoculation period can range from 3 days to 3 months. In primary syphilis, a single, erythematous, painless genital ulcer (called a chancre) appears and usually lasts for 4 weeks. Secondary syphilis may occur weeks to months after resolution of the chancre and is characterized by a variety of physical signs and symptoms. A maculopapular rash on the palms of the hands and soles of the feet is the most recognized physical exam finding in secondary syphilis. Additionally, condyloma lata may appear near the site of the original chancre. These appear as small, verrucous lesions in mucocutaneous areas. Additional symptoms associated with secondary syphilis are fever and lymphadenopathy. Latent stage syphilis can persist for years. This is an asymptomatic stage of disease. Tertiary syphilis is characterized by gummatas, or small, benign growths found throughout various tissues. The patient may also have focal neurologic findings, including hearing and vision loss. Additional findings as a result of syphilis infection include congenital and neurosyphilis. About 90% of syphilis cases occur in men, and of those, 80% occur in men who have sex with men. Risk factors for contracting syphilis include risky sexual practices and male sex. Having syphilis increases the transmissibility of HIV. The presenting history and symptoms of syphilis can be vague. A detailed sexual history should be collected, which includes number of sexual partners, any known exposure to sexually transmitted infections, condom use, and intravenous drug use. The clinical presentation of syphilis depends on the stage of the disease. Syphilis is diagnosed with blood cultures and serology. Two types of tests are appropriate, and due to the possibility of false negatives, both should be performed. A readily available, low-cost nontreponemal test is the Venereal Disease Research Laboratory (VDRL) test. Additionally, a treponemal test should be used as confirmation of disease. These include the fluorescent treponemal antibody absorption (FTA-ABS) or T. pallidum enzyme immunoassay. Intramuscular penicillin G is the treatment of choice regardless of the stage of syphilis, but the stage will determine dosage and frequency of the medication. The prognosis is good for patients with primary syphilis who receive prompt and appropriate treatment. Most patients receive a curative diagnosis after the appropriate course of penicillin. As the disease progresses, complete recovery becomes less likely and prognosis becomes poorer.

Syphilis (ROSH)

Syphilis is a sexually transmitted infection caused by Treponema pallidum, a spirochete that is mainly spread by sexual contact, but can also be transmitted in utero through the placenta, via blood transfusions, and occasionally by direct contact with an active lesion. In the United States, a majority of cases are due to unprotected intercourse and mostly found in men who have sex with men. T. pallidum enters the mucous membranes and can enter the blood and lymphatic system and spread systemically. The disease has four stages: primary, secondary, latent, and tertiary and, if left untreated, the disease will progress through all four stages. Patients with syphilis can present in a variety of ways depending on what stage it is in. The incubation period is approximately 3 weeks long but can vary. Primary syphilis is the earliest stage, characterized by the presence of a highly infectious painless chancre to the genitals. They are solitary, erythematous, raised, and firm to touch and will ulcerate over time, creating a surrounding crater with slightly elevated edges. These lesions usually heal by themselves within 3-6 weeks. There may also be associated regional lymphadenopathy. If the disease is untreated, secondary syphilis will develop about 4-10 weeks after the chancre appears. This stage occurs due to the reproduction of the spirochete and systemic invasion causing a variety of generalized symptoms such as malaise, fever, muscle pain, joint pain, lymph node enlargement, and rash. The rash is nonpruritic and generalized and described as discrete, reddish-brown macules that typically involve the palms and soles. They can also be found on the mucous membranes. Patients may also have condylomata lata (highly infectious grayish-white lesions found in moist areas) and moth-eaten alopecia. Most patients will have resolution of symptoms of secondary syphilis and enter into the latent phase (but will remain seroreactive), while about one-third of patients will develop tertiary syphilis. Latency may last from a few years up to 25 years before the next stage develops. Tertiary syphilis is slowly progressive and mainly affects the cardiovascular system and central nervous system, but it can affect any organ throughout the body. Cardiovascular syphilis develops about 10 years after the primary infection, causing aneurysm formation of the aorta. Neurosyphilis can cause damage to the brain and spinal cord at any stage of the disease causing gait abnormalities, disruption in pain and temperature sensations, memory and speech impairment, psychosis, and progressive dementia. If caught early within the primary or secondary stages, the prognosis of syphilis is very good. However, in patients who develop tertiary syphilis, the recovery depends on the extent of organ damage, which is often irreversible. Diagnosis can be made by microscopic examination and serologic testing. T. pallidum cannot be cultured in a laboratory, and therefore must be identified through direct visualization or detection of clinical specimens. Direct microscopic examination of fresh exudate from moist lesions or from material aspirated from regional lymph nodes can be done by darkfield microscopy. Direct fluorescent antibody (DFA) testing is another test that can be used to identify the organism. Although both tests can help to definitively diagnose this disease, they are only available in select specialized clinics, must be done by a skilled examiner, and the sample should be collected during the early stages of the disease. Therefore, they are considered alternative diagnostic tests in clinical practice. Serologic tests are more commonly used, and there are two types: nontreponemal and treponemal. Either can be used for screening, but confirmatory testing is necessary for both. Nontreponemal tests include the Venereal Disease Research Laboratory (VDRL), rapid plasma reagin (RPR), and Toluidine Red Unheated Serum Test (TRUST) tests. A positive nontreponemal test can be followed up by a confirmatory treponemal test such as the fluorescent treponemal antibody absorption (FTA-ABS) test, microhemagglutination test for antibodies to T. pallidum (MHA-TP), T. pallidum particle agglutination assay (TPPA), T. pallidum enzyme immunoassay (TP-EIA), and chemiluminescence immunoassay (CIA) to rule out false-positive results. Both nontreponemal and treponemal tests should be used for the diagnosis of syphilis, and either can be used as an initial screening test. However, confirmatory testing is necessary due to the potential for false positive test results. Penicillin (penicillin G benzathine 2.4 million units intramuscular) is the drug of choice for the treatment of all stages of syphilis and is the only treatment recommended for neurosyphilis, congenital syphilis, and pregnant women who are affected. Alternative antibiotics include doxycycline, erythromycin, and ceftriaxone but only in penicillin-allergic patients. Patients should be monitored closely with repeat RPR or VDRL at 6- and 12-month intervals to ensure treatment success.

Syphilis (2nd) (ROSH)

Syphilis is a sexually transmitted infection that is caused by the spirochete Treponema pallidum. The incubation period before the onset of symptoms is typically 3 weeks but can range from 3 days to 3 months. The stages of the infection, which determine the clinical features and manifestations, consist of primary, secondary, latent, and tertiary. Secondary syphilis develops over 4 to 8 weeks after the initial painless chancre has resolved. Patients with secondary syphilis classically present with a maculopapular rash affecting the trunk and extremities and often involve the palms and soles. Additionally, condyloma latum (wart-like lesions on the genitals) is seen in patients with secondary syphilis. Patients with secondary syphilis may also have a flu-like illness (headache, sore throat, malaise, or fever), aseptic meningitis, hepatitis, or alopecia. Patients are contagious during this stage and about one-third of untreated individuals develop latent syphilis. The diagnosis of syphilis is made by dark-microscopy (gold standard), which shows the visualization of spirochetes, although this is seldom performed clinically. Serologic tests include nontreponemal tests (rapid plasma reagin, venereal disease research lab), and treponemal tests (fluorescent treponemal antibody absorption, microhemagglutination assay for Treponema pallidum). Nontreponemal tests are ideal for screening, and if positive, should be confirmed using specific treponemal tests. Benzathine penicillin G that is given intramuscularly is the preferred agent for syphilis.

trichomoniasis + metronidazole (ROSH)

The patient has trichomoniasis, a sexually transmitted infection caused by the parasite Trichomonas vaginalis. Trichomonas is a pear-shaped protozoan with four polar flagella that may be seen on wet prep or urine microscopy. It is approximately the same size as a white blood cell. Clinical signs and symptoms associated with trichomoniasis are vaginal pruritus or discomfort, dysuria, and lower abdominal pain. A malodorous, yellow-green, frothy discharge and punctate hemorrhages on the cervix (strawberry cervix) may be seen on speculum exam. Treatment is with a single dose of metronidazole 2 g PO. Patients should avoid alcohol when taking metronidazole and for 48 hours afterward, as it can cause a disulfiram-like reaction (e.g., nausea, vomiting, flushing, dizziness, headache). Another unique side effect of metronidazole is that it can cause a metallic taste in the mouth. Patients with bacterial vaginosis caused by the bacterium Gardnerella vaginalis, should be treated with metronidazole 500 mg BID for 7 days. Night terrors (B) and hallucinations are known side effects of oseltamivir administration for pediatric patients with influenza. Red-orange discoloration of urine (C) is associated with phenazopyridine, a urinary analgesic commonly used for relieving dysuria associated with urinary tract infections. Rifampin, a medication used to treat tuberculosis, can also cause red-orange discoloration of body fluids. Tendon rupture (D) is a side effect of fluoroquinolone antibiotics, particularly in patients > 60 years of age.

trichomoniasis (ROSH)

Vaginitis refers to inflammation or irritation of the vaginal wall and surrounding tissues. Vaginitis has a variety of causes. Noninfectious etiologies include contact allergens or trauma. Infectious etiologies include bacterial vaginosis, candidiasis, gonorrhea, chlamydia, or trichomoniasis. In the case presented in the clinical vignette, a thin, frothy, yellow-green vaginal discharge with associated dysuria and pain with intercourse is most consistent with trichomoniasis. Trichomoniasis is prevalent in 3% of the population of sexually active patients in the United States. It affects both men and women, although men are often asymptomatic. Risky sexual behavior (including multiple sexual partners and unprotected intercourse) is the primary risk factor for trichomoniasis. Patients may present with thin, frothy, yellow-green vaginal discharge that is often malodorous. They may also present with dysuria, pain with intercourse, and lower abdominal pain. Physical exam will reveal a thin, malodorous, purulent, green or yellow vaginal discharge. Vaginal edema, erythema, and punctate macular hemorrhages on the cervix, known as colpitis macularis or strawberry cervix, may also be seen. A wet mount is the diagnostic test most commonly used. A vaginal smear is placed on a glass slide and viewed under microscopy. If Trichomonas vaginalis (flagellated protozoa moving in corkscrew patterns, shaking, or rotating) are observed, the wet mount is considered diagnostic for trichomoniasis. If this test result is negative and trichomoniasis is still suspected, nucleic acid amplification testing may be performed. The treatment for trichomoniasis is oral metronidazole 500 mg by mouth twice a day for 7 days. All sexual partners should receive treatment as well. Repeat testing (by either vaginal wet mount or nucleic acid amplification testing) should be performed 2 weeks to 3 months after antibiotic completion to ensure curative therapy. During this time, sexual abstinence is strongly encouraged. The potential complications for women with trichomoniasis include urethritis, cystitis, pelvic inflammatory disease, and infertility. Potential complications for men with trichomoniasis include prostatitis, epididymitis, infertility, and prostate cancer. In both men and women, the risk of HIV transmission is increased with trichomoniasis infection.

Vulvovaginal candidiasis (ROSH)

Vaginitis refers to inflammation or irritation of the vaginal wall and surrounding tissues. Vaginitis has a variety of causes. Noninfectious etiologies include contact allergens or trauma. Infectious etiologies include bacterial vaginosis, candidiasis, gonorrhea, chlamydia, or trichomoniasis. In this case, a thick, white vaginal discharge with associated vulvar pruritus and burning is most consistent with candidiasis. Vulvovaginal candidiasis is common. Up to 75% of women in the United States report a history of vulvovaginal candidiasis in the last year. After bacterial vaginosis, it is the second most common cause of vaginitis. Vulvovaginal candidiasis is caused by an overgrowth of Candida species. Candida may be found naturally in up to 25% of women. Most vulvovaginal candidiasis is caused by C. albicans, and the remainder of cases are caused by C. glabrata. C. glabrata presents a unique challenge due to its resistance to azole therapy. Risk factors for vulvovaginal candidiasis include a history of sexually transmitted infections, recent antibiotic use, diabetes, vaginal douching, risky sexual practices, and pregnancy. Pruritus is the most common concern for women with vulvovaginal candidiasis. Patients may also report burning, cutaneous erythema, and pain with intercourse. Excoriations may be present due to intense pruritus. Satellite lesions (small, erythematous, well-demarcated patches with scalloped edges) are common. Vaginal discharge is typically white and thick (like curds or cottage cheese). It has no odor, and the pH is normal (4.0 to 4.5). The diagnosis can be made with history and physical exam or by testing the vaginal pH. The microscopic evaluation of a vaginal sample using either saline or potassium hydroxide will reveal hyphae and spores. This is sometimes referred to as spaghetti and meatballs for the shape and distribution of the fungal components. The first-line therapy for vulvovaginal candidiasis is fluconazole 150 mg taken orally in one dose. There are over-the-counter antifungal formulations that are available for patients who have a previous diagnosis of vulvovaginal candidiasis and elect topical therapy. These include miconazole, tioconazole, and clotrimazole.

Vaginitis + RF (ROSH)

Vaginitis refers to inflammation or irritation of the vaginal wall and surrounding tissues. Vaginitis has a variety of causes. Noninfectious etiologies include contact allergens or trauma. Infectious etiologies include bacterial vaginosis, candidiasis, gonorrhea, chlamydia, or trichomoniasis. In this case, a thin, homogenous, vaginal discharge with a fishy odor is most consistent with bacterial vaginosis. Bacterial vaginosis is the most common cause for vaginal discomfort and malodorous vaginal discharge. It is estimated to affect 30-50% of women. The pathophysiology of bacterial vaginosis involves Gardnerella vaginalis and a disruption in the normal microbiome of the vagina and decreased amounts of lactobacilli in the vagina. The microbiome then is replaced by Gardnerella vaginalis and causes a change in vaginal pH from a more acidic environment to a more basic environment. Gardnerella vaginalis and other pathogenic bacteria (including Prevotella and Mobiluncus species) thrive in a basic environment making treatment difficult. Risk factors for bacterial vaginosis include multiple sexual partners, risky sexual behavior, smoking, and douching regularly. It is less common in women using oral estrogen-containing contraception. Patients with bacterial vaginosis often present with a thin, white vaginal discharge with a foul or fishy odor. There are usually no other presenting symptoms, and most patients presenting with bacterial vaginosis are completely asymptomatic. Physical exam typically reveals a thin, milky discharge with a fishy odor. Bimanual exam usually has normal findings, and there is no cervical motion tenderness. A Gram stain will definitively diagnose bacterial vaginosis. If unavailable, Amsel criteria may be used in place of a Gram stain. Bacterial vaginosis can be diagnosed if three of the four Amsel criteria are satisfied: thin, homogeneous discharge; clue cells on microscopy or wet mount; a vaginal pH > 4.5; and a fishy, malodorous discharge with or without the whiff test. The whiff test refers to the olfactory evaluation of vaginal discharge when mixed with potassium hydroxide. The test is considered positive if a fishy odor is produced. Often, the odor may be appreciated without the addition of potassium hydroxide. First-line therapy for bacterial vaginosis is metronidazole 500 mg oral twice daily for 7 days. Vaginal preparations are also available, including 0.75% metronidazole gel intravaginally for 5 days. In pregnant women, bacterial vaginosis may be associated with spontaneous abortion and preterm delivery. Therefore, proper identification, diagnosis, and treatment are imperative. The risk of bacterial vaginosis is low, and the prognosis is typically very good. Unfortunately, patients with a history of bacterial vaginosis are more likely to have recurrent infections despite prevention. Up to 50% of patients will have a recurrence within 12 months.

Vulvovaginal atrophy (ROSH)

Vulvovaginal (urogenital) atrophy involves atrophy to the vagina, labia, and other female urogenital structures due to a decline in serum estrogen levels. It occurs most often in menopausal women but may occur in hypoestrogenic states related to medications, such as gonadotropin-releasing hormone agonists (leuprolide and nafarelin) and selective estrogen receptor modulators. The classic symptoms and signs include vulvovaginal dryness, burning, and irritation; urinary symptoms, such as dysuria, urinary frequency, and recurrent urinary tract infections; and reduced lubrication during sexual intercourse causing dyspareunia. The supportive physical exam findings include tissue fragility or fissures, vulvovaginal pallor, vulvovaginal erythema, loss of vaginal rugae, and decreased vulvovaginal secretions. The course of vulvovaginal atrophy is often progressive and worsens as the duration of the low estrogen state persists. The diagnosis of vulvovaginal atrophy is made clinically. Microscopy of vaginal discharge and urinalysis may be performed if there is concern for vaginal infection or urinary tract infection. Laboratory testing to confirm a hypoestrogenic state is not necessary. Women with vulvovaginal atrophy should be treated when the symptoms are bothersome and causing distress. It is important to ask patients near menopause about these symptoms during routine wellness visits because many women will not report these complaints to their health care clinician. The initial treatment for women with vaginal dryness and dyspareunia is vaginal moisturizers (two or three times per week) and lubricants (during intercourse). However, most women will require hormonal therapy. Hormonal therapy may be used for patients without contraindications who do not have adequate relief with vaginal moisturizers and lubricants. Vaginal estrogen is generally preferred because it is more efficacious than other hormonal options and is generally dosed less frequently. The other hormonal treatment options include vaginal dehydroepiandrosterone, ospemifene (oral selective estrogen receptor modulator), and the off-label use of topical testosterone. Vaginal estrogen does not usually have to be opposed by an oral progestin and is generally thought to carry little risk of causing endometrial hyperplasia or endometrial cancer. Contraindications to its use include prior or current estrogen-dependent tumors. Women with severe changes may require pelvic physical therapy and vaginal dilators. Laser-based therapy exists but the efficacy and safety remains uncertain.

Vulvovaginal candidiasis + MC Sx (ROSH)

Vulvovaginal candidiasis is a yeast infection of the vulva and the vagina. The most common symptom is vulvar pruritus and other common symptoms include vulvar burning, soreness, or irritation. Patients may have dysuria or dyspareunia. The dysuria associated with vulvovaginal candidiasis is classically described as external or vulvar rather than urethral, as in cystitis. Physical examination findings frequently include erythema, excoriations, or fissures of the vulva and the vagina. Patients with vulvovaginal candidiasis may or may not have vaginal discharge, and it is classically a white, thick, and clumpy discharge when it is present. The symptoms and signs of vulvovaginal candidiasis vary from mild to severe. Symptoms may be worse the week prior to menses. The diagnosis of vulvovaginal candidiasis is confirmed by detecting Candida on a wet mount, Gram stain, or culture of vaginal discharge. The diagnosis should not be made solely based on clinical findings because there are no pathognomonic findings. The recommended treatment for vulvovaginal candidiasis is topical or oral antifungal therapy for symptomatic patients. Asymptomatic women and male sexual partners of infected women do not require treatment. The best treatment regimen varies depending on whether the infection is uncomplicated or complicated. Vulvovaginal candidiasis is uncomplicated when the symptom severity is mild to moderate and it is nonrecurrent, caused by Candida albicans, and occurs in an immunocompetent host. Patients with uncomplicated infection are treated with oral fluconazole administered as a single oral dose. Patients with vulvovaginal candidiasis and uncontrolled diabetes mellitus are considered to have complicated infections. Patients with complicated infection are treated with two doses of oral fluconazole administered as oral doses 3 days apart.

condyloma acuminatum

genital warts caused by HPV

Condyloma

infection of the genitals

Condyloma lata

white lesions associated with secondary syphilis

For each of the following conditions, students should describe the underlying pathophysiology, presenting signs and symptoms, basic epidemiology, differential diagnosis, associated systemic disorders (if indicated), appropriate diagnostic studies, clinical intervention, pharmaceutical therapies, and health maintenance concerns: Pelvic Inflammatory disease

•Ascending infection of the upper reproductive tract. (cervix or vagina to endometrium or uterine tubes) ETIOLOGIES •Usually mixed - Chlamydia trachomatis (most common), Neisseria gonorrhoeae, Gardnerella vaginalis, M. genitalium, anaerobes, enteric or respiratory pathogens etc. RISK FACTORS •Multiple sex partners, unprotected sex, BV, douching, prior PID, age 15-19, nulliparous, IUD placement, c-section or I&D. CLINICAL MANIFESTATIONS •Bilateral Pelvic or lower abdominal pain, dysuria, dyspareunia, mucopurulent vaginal discharge or bleeding, nausea, vomiting. •Physical examination: lower abdominal tenderness, fever, purulent cervical discharge. Cervical motion tenderness (chandelier sign). DIAGNOSIS •Primarily clinical - abdominal tenderness + cervical motion tenderness + adnexal tenderness + plus at least 1 of the following: positive gram stain, temperature >38 C, WBC >10,000, pus on culdocentesis or laparoscopy, pelvic abnormality on bimanual exam or ultrasound, increased ESR or CRP. •Workup includes pregnancy test (rule out Ectopic pregnancy) & nucleic acid amplification test (for Gonorrhea and Chlamydia). •US = enlarged fallopian tubes with cul-de-sac fluid •Laparoscopy is the most accurate test for PID (rarely performed) - may be done in uncertain cases, severe disease, or if no improvement with antibiotics to rule out ectopic or appendicitis OUTPATIENT MANAGEMENT •Ceftriaxone (500mg IM x 1 dose) PLUS Doxycycline (100mg bid x 14 days). Metronidazole 500mg bid × 14 days often added. •Levofloxacin + Metronidazole alternative if true Penicillin allergy. INPATIENT MANAGEMENT •Second generation cephalosporin (eg, Cefoxitin or Cefotetan) + IV Doxycycline. •Clindamycin + Gentamicin is an alternative (eg, pregnancy or true Penicillin allergy).

Asymptomatic bacteriuria

•Asymptomatic patient with incidental bacteriuria on urinalysis. •No treatment needed: general population, elderly, diabetics, nonpregnant premenopausal women, spinal cord injury patients or patients with chronic indwelling urinary catheters. •Treatment needed: pregnancy, patients with urologic intervention & hip arthroplasty. •Pregnancy requires screening & treatment because it can be associated with pre-term birth, perinatal death, & pyelonephritis in the mother.

For each of the following conditions, students should describe the underlying pathophysiology, presenting signs and symptoms, basic epidemiology, differential diagnosis, associated systemic disorders (if indicated), appropriate diagnostic studies, clinical intervention, pharmaceutical therapies, and health maintenance concerns: Candidiasis

•Candida albicans overgrowth (part of the normal flora due to change in normal vaginal environment (ex. use of abx) •increased risk = DM, steroid, pregnancy, ABX use, OCP and diaphragm use •Vaginal & vulvar erythema, swelling, burning, pruritus •Burning when urine touches skin, Dysuria, Dyspareunia •THICK CURD-LIKE/COTTAGE CHEESE DISCHARGE Normal pH (acidic) and no odor •HYPHAE, YEAST* & spores on KOH prep •FLUCONAZOLE (PO x 1 dose) •Intravaginal antifungals -Clotrimazole, Nystatin -Butoconazole -Miconazole •Keep vagina dry, 100% cotton underwear, avoid tight-fitting clothes, avoid use of feminine deodorants & bubble baths

For each of the following conditions, students should describe the underlying pathophysiology, presenting signs and symptoms, basic epidemiology, differential diagnosis, associated systemic disorders (if indicated), appropriate diagnostic studies, clinical intervention, pharmaceutical therapies, and health maintenance concerns: Cervicitis - Chlamydia

•Chlamydia trachomatis is the most common overall bacterial cause of STIs in the US. CLINICAL MANIFESTATIONS •Urethritis: purulent or mucopurulent discharge, pruritus, dysuria, dyspareunia (pain with intercourse), post-coital bleeding, hematuria. Up to 40% asymptomatic (especially men). •Pelvic inflammatory disease: abdominal pain, positive cervical motion tenderness. •Reactive arthritis; urethritis, uveitis, arthritis (reactive arthritis is an autoimmune reaction). positive HLA-B27. •Lymphogranuloma venereum: genital/rectal lesion with softening, suppuration & lymphadenopathy. DIAGNOSIS •Nucleic acid amplification most sensitive & specific for C. trachomatis, N. gonorrhoeae, & M. genitalium (vaginal swab or first-catch urine preferred). •Genetic probe methods, culture, antigen detection. MANAGEMENT •Azithromycin (1 gram x 1 dose; if pregnant) or Doxycycline 100mg bid for 10 days. Re-test in 3 weeks to ensure clearance of the organism. •Also treat for Gonorrhea (Ceftriaxone 500mg IM × 1 dose). COMPLICATIONS PID, infertility with chronic pelvic pain, salpingitis, toa, Fitz-Hugh Curtis, ectopic pregnancy

For each of the following conditions, students should describe the underlying pathophysiology, presenting signs and symptoms, basic epidemiology, differential diagnosis, associated systemic disorders (if indicated), appropriate diagnostic studies, clinical intervention, pharmaceutical therapies, and health maintenance concerns: Syphilis

•Chronic infection caused by the spirochete Treponema pallidum. •Known as "the great imitator" because the rash & disease can present in many different ways similar to other diseases. TRANSMISSION •Direct contact of a mucocutaneous lesion (eg, sexual activity). May also be transmitted to the fetus via the placenta. PATHOPHYSIOLOGY •T. pallidum enters tissues from direct contact, forming a chancre at the inoculation site and from there, goes to the regional lymph nodes before disseminating. CLINICAL MANIFESTATIONS Primary Syphilis •Chancre: painless ulcer at or near the inoculation site with raised indurated edges (usually begins as a papule that ulcerates). Chancres heal spontaneously on average within 3-4 weeks (even without medical management). •Nontender regional lymphadenopathy near the chancre site lasting 3-4 weeks. Secondary Syphilis Symptoms may occur a few weeks - 6 months after the initial symptoms. •Maculopapular rash diffuse bilateral maculopapular lesions (involvement of the palms & soles common). Lesions may be pustular in some patients. •Condyloma lata: wart-like, moist lesions involving the mucous membranes & other moist areas. Especially near the chancre site. Highly contagious. •Systemic symptoms: fever, lymphadenopathy (may be tender), arthritis, meningitis, headache, hepatitis (elevated alkaline phosphatase), alopecia. Tertiary (late) Syphilis May occur from 1 to > 20 years after initial infection or after latent infection. •Gumma: noncancerous granulomas on skin & body tissues (eg, bones). •Neurosyphilis: headache, meningitis, dementia, vision/hearing loss, incontinence; Tabes dorsalis (demyelination of posterior columns leading to ataxia, areflexia, burning pain, weakness). •Argyll-Robertson pupil: small, irregular pupil that constricts with accommodation but is not reactive to light. •Cardiovascular: aortitis, aortic regurgitation, aortic aneurysms. DIAGNOSIS Nontreponemal tests (screening): •RPR (Rapid Plasma Reagin). These tests look at titers (eg, a positive test indicates a titer of 1:32 or greater). Because they are nonspecific, a positive RPR or VDRL test must be confirmed by specific treponemal testing (eg, FTA-ABS). RPR is usually positive 4-6 weeks after infection. Changes in titers also help to determine therapeutic response. •VDRL: Venereal Disease Research Laboratory. •Nontreponemal tests are nonspecific. False positives can be seen with antiphospholipid syndrome, pregnancy, tuberculosis, Rickettsial infections (eg, Rocky Mountain spotted fever). Treponemal testing (confirmatory): •FTA-ABS (fluorescent treponemal antibody absorption) •Microhemagglutination test for T. pallium antibodies. Darkfield microscopy •Allows for direct visualization of T. pallidum from chancre or condyloma lata. MANAGEMENT •Penicillin is the treatment of choice for all stages of Syphilis. Early Syphilis: (primary, secondary & early latent) •Penicillin G benzathine 2.4 million units IM x1 dose. •Doxycycline for 14 days if Penicillin-allergic. Late Syphilis: (tertiary or late latent) •Penicillin G benzathine 2.4 million units IM once weekly x 3 weeks. Neurosyphilis: •IV Penicillin G preferred (3-4 million units every 4 hours for 10 - 14 days). •Patients with Penicillin allergy who are pregnant, present with neurosyphilis, have cardiovascular manifestation of late syphilis, or have treatment failure should be tested for Penicillin allergy and desensitized (if immediate-type reaction) or rechallenged (if delayed reaction) with Penicillin MONITORING •All patients with Syphilis should be tested for HIV and other STIs. •All patients should be reexamined clinically & serologically at 6 months and 12 months after treatment. Patients with HIV may be monitored more frequently. •A nontreponemal tier should be obtained prior to initiating therapy. A fourfold decline in the nontreponemal tier within 6 months is considered an acceptable response JARISCH-HERXHEIMER REACTION •An acute, self-limited febrile reaction that usually occurs within the first 24 hours after receiving therapy for a spirochetal infection (eg, Syphilis, Lyme disease). •It is thought to be due to the release of cytokines and immune complexes from killed organisms. •It classically presents with fever, chills, headache, myalgias, hypotension, and worsening of the rash may occur. Management •Is self-limited and usually resolves without intervention in 12 to 24 hours but NSAIDS or antipyretics may be used for symptoms.

For each of the following conditions, students should describe the underlying pathophysiology, presenting signs and symptoms, basic epidemiology, differential diagnosis, associated systemic disorders (if indicated), appropriate diagnostic studies, clinical intervention, pharmaceutical therapies, and health maintenance concerns: Lymphogranuloma venereum

•Genital ulcer disease cause by L1, L2 & L3 serovars of Chlamydia trachomatis. •Most commonly seen in tropical & subtropical areas of the world. •Pathophysiology: extension from the infection site to the draining lymph nodes. CLINICAL MANIFESTATIONS •Painless genital ulcer at the site of inoculation. •The secondary stage appears 2-6 weeks later - painful inguinal &/or femoral lymphadenopathy (buboes). •May develop proctocolitis (rectal discharge, anal pain, constipation, fever and/or tenesmus). DIAGNOSIS •Often clinical. •Cultures and serologic testing have low yield. Nuclei acid amplification testing. MANAGEMENT •Doxycycline 100mg bid x 21 days treatment of choice. •Azithromycin 1g orally once weekly × 3 weeks. •Buboes may need needle aspiration or incision & drainage to avoid rupture or sinus tract formation. •All patients should be tested for other STIs including HIV.

Pyelonephritis

•Infection of the upper genitourinary tract (kidney parenchyma & renal pelvis). PATHOPHYSIOLOGY •Usually an ascending infection from the lower urinary tract. ETIOLOGIES •E. coli most common (>80%), other gram-negative uropathogens (eg, Proteus, Enterobacter, Klebsiella & Pseudomonas). Enterococci with indwelling catheters. RISK FACTORS •Diabetes mellitus, history of recurrent UTIs or kidney stones, pregnancy, congenital urinary tract malformations. CLINICAL MANIFESTATIONS •Upper tract symptoms: fever, chills, back or flank pain. Nausea & vomiting not common but suggestive. •Lower tract symptoms: dysuria, urgency, & frequency. PHYSICAL EXAMINATION •Positive costovertebral angle tenderness, fever, & tachycardia. DIAGNOSIS •UA: Pyuria [>10 WBCs/hpf], + leukocyte esterase, +Nitrites (90% bacteria causing UTIs), hematuria, cloudy urine, bacteriuria. WBC casts hallmark of Pyelonephritis. Increased pH with Proteus. •CBC: leukocytosis with left shift. •Urine culture definitive diagnosis. OUTPATIENT MANAGEMENT •Fluoroquinolones first-line (if resistance rate < 10%). •If resistance > 10%, initiate either IV Ceftriaxone or Gentamicin followed by an oral Fluoroquinolone. INPATIENT MANAGEMENT •Third- or fourth-generation Cephalosporins, Fluoroquinolones, Aminoglycosides, or extended- spectrum Penicillins total duration 2 weeks. -Indications for admission include older age, signs of obstruction, comorbid conditions, or inability to tolerate oral antibiotics. PREGNANCY •IV Ceftriaxone first-line. Aztreonam if Penicillin-allergic. •Avoid Trimethoprim-Sulfamethoxazole, Aminoglycosides, Fluoroquinolones, & Tetracyclines.

For each of the following conditions, students should describe the underlying pathophysiology, presenting signs and symptoms, basic epidemiology, differential diagnosis, associated systemic disorders (if indicated), appropriate diagnostic studies, clinical intervention, pharmaceutical therapies, and health maintenance concerns: Cervicitis - Gonorrhea

•Neisseria gonorrhoeae (gram-negative diplococci). 2-8 day incubation period. CLINICAL MANIFESTATIONS •Urethritis and Cervicitis; discharge*** (anal, vaginal, or pharyngeal), Pelvic inflammatory disease DISSEMINATION •Triad of dermatitis, polyarthralgias, & tenosynovitis - rash (maculopapular, petechial), arthralgia (joint pain), tenderness along the tendon sheath. Often associated with fever, chills, & malaise. •Purulent gonococcal septic arthritis (especially the knee). In women, it occurs more frequently during menses. DIAGNOSIS •Nucleic acid amplification most sensitive & specific for C. trachomatis, N. gonorrhoeae, & M. genitalium (recommended over culture). -First-void or first-catch urine ideal if urethritis. -If disseminated, samples are taken at multiple sites (eg, urethral, rectal, pharyngeal, cervical). •Gram stain: 2 or more WBCs/hpf. No organisms seen is suggestive of Nongonococcal urethritis (eg,Chlamydia trachomatis). Gram-negative diplococci = N. gonorrhoeae. Urethral swab. •Urinalysis or dipstick: positive leukocyte esterase on dipstick or 10 or more WBCs/hpf (pyuria) on microscopy suggestive. •Synovial fluid: nucleic acid amplification testing (NAAT) or culture on chocolate agar or Thayer-Martin medium. •Nucleic acid amplification tests taken at multiple sites (eg, urethral, rectal, pharyngeal, cervical). •Blood cultures. MANAGEMENT: •Gonococcal arthritis: IV Ceftriaxone is the first-line treatment. •Urethritis & Cervicitis: Ceftriaxone 500mg IM plus either Doxycycline or Azithromycin if pregnant (for additional coverage for Gonorrhea as well as to cover possible Chlamydia). COMPLICATIONS PID, infertility with chronic pelvic pain, salpingitis, toa, Fitz-Hugh Curtis

Urethritis/Cervicitis

•Nongonococcal urethritis (NGU): Chlamydia trachomatis is the most common cause of nongonococcal urethritis. Chlamydia trachomatis is also the most common bacterial STI in the US. 5-8 days incubation period followed by purulent or mucopurulent discharge. May be associated with pruritus, hematuria, or dyspareunia. Up to 40% asymptomatic. -Others include Ureaplasma urealyticum, Trichomonas vaginalis, M. genitalium, & viruses. •Gonococcal urethritis: abrupt onset of symptoms (especially within 3-4 days). Opaque, yellow, white or clear thick discharge, pruritus. Up to 20% of patients are asymptomatic. CLINICAL MANIFESTATIONS •Urethral discharge & vaginal pruritus. •Dysuria seen in both gonococcal & nongonococcal urethritis. •Abdominal pain or abnormal vaginal bleeding. DIAGNOSIS •Nucleic acid amplification most sensitive & specific for C. trachomatis, N. gonorrhoeae & M. genitalium (recommended over culture). First-void or first-catch urine ideal. •Gram stain: 2 or more WBCs/hpf. No organisms seen suggestive of NGU. Gram-negative diplococci = N. gonorrhoeae. Urethral swab. •Urinalysis or dipstick: positive leukocyte esterase on dipstick or 10 or more WBCs/hpf (pyuria) on microscopy suggestive. MANAGEMENT •25-30% of patients have co-infection of both Gonorrhea & Chlamydia so empiric treatment of both is recommended if test results are not available. Based on testing: •N. gonorrhoeae: Ceftriaxone 500mg IM x 1 dose PLUS Azithromycin 1g x 1 dose (for additional coverage for Gonorrhea due to increased resistance as well as to cover possible Chlamydia). •Chlamydia: Azithromycin 1g orally (single observed dose) or Doxycycline 100mg orally bid x 10 days.

For each of the following conditions, students should describe the underlying pathophysiology, presenting signs and symptoms, basic epidemiology, differential diagnosis, associated systemic disorders (if indicated), appropriate diagnostic studies, clinical intervention, pharmaceutical therapies, and health maintenance concerns: Bacterial vaginosis

•Overgrowth of Gardnerella vaginalis & anaerobes (eg, Mobiluncus, Peptostreptococcus) due to altered biome - decreased Lactobacillus acidophilus (L. acidophilus normally maintains vaginal pH). •Although it is not a sexually transmitted infection, it is more common in sexually active women with new or multiple partners (changes the vaginal biome). CLINICAL MANIFESTATIONS •Vaginitis: malodorous vaginal discharge worse after sex, vaginal itching, burning, & dyspareunia. DIAGNOSIS •Amsel criteria: copious, thin, homogenous, grayish-white vaginal discharge, vaginal pH >4.5, positive whiff-amine test - "fishy odor" when a drop of 10% KOH is added, clue cells on saline wet mount (epithelial cells covered by bacteria), few WBCs (not inflammatory), & few lactobacilli. MANAGEMENT •Metronidazole x 7 days (gel or oral) or Clindamycin (oral or gel). Both are safe in pregnancy. •Partners do not need treatment. Treatment for asymptomatic nonpregnant women not indicated. PREVENTION; avoid douching & smoking. •Pregnancy complications include Prelabor rupture of membranes, chorioamnionitis, & preterm labor.

Acute cystitis

•Pathophysiology: usually an ascending infection of the lower urinary tract from the urethra. RISK FACTORS •Women; sexual intercourse "honeymoon cystitis", spermicidal use (especially with diaphragm). •Pregnancy: progesterone & estrogen causes ureter dilation & inhibition of bladder peristalsis. •Elderly & Postmenopausal, Diabetes mellitus & presence of an indwelling catheter. In children, may indicate vesicourethral reflux. •Infants (should receive a bladder and renal ultrasound to rule out congenital abnormalities). •Complicated: underlying condition with risk of therapeutic failure: symptoms >7 days, pregnancy, diabetics, immunosuppression, indwelling catheter, anatomic abnormality, elderly, males. ETIOLOGIES •Escherichia coli most common (>80%), •Staphylococcus saprophyticus 2nd most common in sexually active women. •Other gram-negative uropathogens - Klebsiella, Proteus, Enterobacter, & Pseudomonas. •Enterococci with indwelling catheters. CLINICAL MANIFESTATIONS •Irritative symptoms: dysuria (burning), frequency, & urgency. •Hematuria, suprapubic pain, & tenderness DIAGNOSIS •Urinalysis: pyuria (>10 WBCs/hpf), hematuria, leukocyte esterase; nitrites, cloudy urine, bacteriuria, increased pH with Proteus. •Urine culture: definitive diagnosis. Women: ≥1,000 CFU/ml of uropathogens - clean catch specimen. Epithelial (squamous cells) = contamination. -Indications for culture: complicated UTI, infants or children, elderly, males, urologic abnormalities, refractory to treatment, or catheterized patients. MANAGEMENT OF UNCOMPLICATED •1st-line: Nitrofurantoin or Trimethoprim-sulfamethoxazole or Fosfomycin (if resistance pattern < 20%) •2nd-line: Fluoroquinolones (may be used first line if sulfa allergies or increased resistance patterns). Cephalosporins or Cefpodoxime. ADJUNCT •Increase fluid intake, void after intercourse, Hot sitz baths may provide relief of abdominal discomfort. •Phenazopyridine is a bladder analgesic not used more than 48h due to side effects (methemoglobinemia, hemolytic anemia). Inform the patient Phenazopyridine turns the urine an orange color. COMPLICATED •Fluoroquinolones PO or IV, Aminoglycosides x7-10 or 14 days (depending on the severity). CYSTITIS DURING PREGNANCY •Amoxicillin, Amoxicillin-clavulanate, Cephalexin, Cefpodoxime, Nitrofurantoin, & Fosfomycin. Sulfisoxazole is safe except in last days of pregnancy (can lead to kernicterus). •Avoid Trimethoprim-sulfamethoxazole in the first trimester, Aminoglycosides, Fluoroquinolones, and Doxycycline.

Fitz-Hugh Curtis Syndrome

•Perihepatitis with hepatic fibrosis, scarring & peritoneal surface of the anterior right upper quadrant in the setting of Pelvic inflammatory disease (PID). •Seen in 10% of women with PID CLINICAL MANIFESTATIONS •RUQ pain due to perihepatitis (liver capsule involvement). May radiate to the right shoulder. PHYSICAL EXAMINATION •Marked RUQ tenderness. DIAGNOSIS •Laparoscopy - "violin-string" adhesions on the anterior liver surface. Often have normal LFTs (or slight elevations).

For each of the following conditions, students should describe the underlying pathophysiology, presenting signs and symptoms, basic epidemiology, differential diagnosis, associated systemic disorders (if indicated), appropriate diagnostic studies, clinical intervention, pharmaceutical therapies, and health maintenance concerns: Atrophic vaginitis

•Seen in hypoestrogenic states (eg. menopause, postpartum lactation, and postpartum women, progesterone-only or low-dose oral contraceptives) . CLINICAL MANIFESTATIONS •Vaginal dryness, dyspareunia, vaginal inflammation, infection & recurrent UTIs with increased pH (loss of lactobacilli which normally converts glucose to lactic acid). MANAGEMENT •Vaginal moisturizers: improves symptoms (ex. dyspareunia, dryness) but no effect on atrophy. •Topical vaginal estrogens safest, most effective medical therapy - cream, vaginal ring, vaginal troches. Adverse effects: vaginal bleeding, breast pain, nausea, thromboembolism (CVA, DVT, PE), endometrial cancer. Less risk compared to oral estrogen. Estrogen increases hepatic production of coagulation factors. •Ospemifene: selective estrogen receptor modulator (SERM) that is an estrogen agonist in the vagina & bone and an estrogen antagonist in the breast & uterus.

For each of the following conditions, students should describe the underlying pathophysiology, presenting signs and symptoms, basic epidemiology, differential diagnosis, associated systemic disorders (if indicated), appropriate diagnostic studies, clinical intervention, pharmaceutical therapies, and health maintenance concerns: Chancroid

•Sexually transmitted infection leading to genital ulcers, lymphadenopathy, and bubo formation. •Etiology: Haemophilus ducreyi transmitted after a break in the skin. Gram-negative fastidious coccobacillus. •Incubation period 3-7 days. •Risk factors: most common in children and young adults. •10% confection with HV or Syphilis in the US (rare in US). CLINICAL MANIFESTATIONS •One or more painful genital ulcers at the inoculation site (well-defined irregular borders that are sometimes undermined on an erythematous base & may be covered with a gray or yellow purulent exudate) followed by painful inguinal lymphadenopathy, which can liquefy and become fluctuant (bubo formation). •Dysuria and dyspareunia DIAGNOSIS •Usually a clinical diagnosis (difficult to culture). •PCR or immunochromatography. •Must rule out HSV by (PCR or culture) and rule out Syphilis (RPR and VRDL). MANAGEMENT •Azithromycin 1g x 1 dose. •Ceftriaxone 250mg IM x 1; Erythromycin 500mg tid x 7 days; Ciprofloxacin.

For each of the following conditions, students should describe the underlying pathophysiology, presenting signs and symptoms, basic epidemiology, differential diagnosis, associated systemic disorders (if indicated), appropriate diagnostic studies, clinical intervention, pharmaceutical therapies, and health maintenance concerns: Trichomoniasis

•Trichomonas vaginalis is a flagellated protozoan that is transmitted sexually. CLINICAL MANIFESTATIONS •Women: vaginitis, cystitis, or cervicitis - copious malodorous vaginal discharge worse with menses, postcoital bleeding, dyspareunia, dysuria, & frequency. PHYSICAL EXAMINATION •Copious frothy yellow-green vaginal discharge, vulvovaginal erythema. •Cervical petechiae (strawberry cervix). DIAGNOSIS •Microscopic examination (saline wet mount) - mobile protozoan trophozoites, vaginal pH >4.5, increased WBCs. •Nucleic acid amplification test (NAAT) or culture should be performed if wet mount is negative. MANAGEMENT •Metronidazole 2g oral dose x1 dose or 500mg bid x 7 days. Tinidazole alternative. Treatment is indicated for both symptomatic & asymptomatic men & women. Partners must be treated. •Follow up: because of high rate of reinfection, retesting with NAAT performed within 3 months of initial treatment. •Reinfection: Metronidazole 500mg orally bid x 7 days (single-dose therapy should be avoided if recurrent). COMPLICATIONS •Increased risk of HIV transmission, complications during pregnancy.


Related study sets

General Biology 1011 Exam 3 Review

View Set

Comprehensive MSK Review (all classes)

View Set

AP Spanish Lang & Cult: Cortometraje Ella y Yo

View Set